The group of decimals in ascending order is:
A 261,2; 261,3; 261,342; 261,4
B. 261,2; 261,3; 261,4; 261,342
С 261,342; 261,4; 261,3; 261,2
D 261,4; 261,342; 261,3; 261,2
E 261,03; 261, 30; 261,33; 261,003​

Answers

Answer 1

Answer:

A

Step-by-step explanation:

it's just going from least to greatest


Related Questions

If AB = 4 centimeters, which is closest to BC?

Options
4.7 cm
3.4 cm
2.5 cm
2.1 cm

Answers

in my opinion it is 3.4 cm because in any case it is a right triangle,then you can see whith the naked eye that AB is longer

A figure has vertices at (-3,5), (-1,8), (1,5), (1,1), (-1,-2) and (-3,1). What is the correct name
of this figure?

Answers

Answer:

regular hexagon

Step-by-step explanation:

find three rational numbees between -2/5 and -1/5​

Answers

Answer:

-3/10, -25/100, -3/8

A rectangular gate is made using 6 straight pieces of steel.the weight of the steel is 2.5 kg per metre . Work out the total weight of the steel gare

Answers

Answer:

150 kg

Step-by-step explanation:

here is the complete question

A rectangular gate is made using 6 straight pieces of steel.

5 m

12 m

The weight of the steel is 2.5 kg per metre.

Work out the total weight of the steel gate.

To solve this question determine the square root of 12 and 5

√169 = 13

5 + 12+ 5 + 12+ 13 + 13 = 60 m

total weight = 60 x 2.5 = 150kg

Need help with these questions

Answers

Answer:

Area = 31.5 units²

Step-by-step explanation:

Area of the triangle = ½ × base × height

base = 9

height = 7

Substitute

Area = ½ × 9 × 7

Area = ½ × 63

Area = 31.5 units²

Provide one reason for when you can use the distance formula.

Answers

Answer:

Draw a right triangle such that the distance between the two points is the hypotenuse. Hence, When you use the distance formula, you are calculating the length of the Hypotenuse of a right triangle.

Please help me with this it’s confusing ,

Answers

Answer:

It’s the 4th one

Step-by-step explanation:

I believe that number four is the correct answer :)

julie wants to invest $3,000 into
a mutual fund that pays 7%
interest for 10 years. suppose
the interest were compounded
monthly instead of annually.
how much would the future
value of the investment
increase?

Answers

Answer:

$400

Step-by-step explanation:

The future value of the investment increase will be $6,028.98.

What is compound Interest?

The interest that is calculated using both the principal and the interest that has accrued during the previous period is called compound interest. It differs from simple interest in that the principal is not taken into account when determining the interest for the subsequent period with simple interest. Compound interest is commonly abbreviated C.I. in mathematics.

Given:

P = $3000

R = 7%

T = 10 years

r = R/100

r = 7/100

r = 0.07 rate per year,

Then solve the equation for A

A = P[tex](1 + r/n)^{nt[/tex]

A = 3,000.00[tex](1 + 0.07/12)^{(12)(10)[/tex]

A = 3,000.00[tex](1 + 0.0058333333333333)^{(120)[/tex]

A = $6,028.98

Hence, the future value will be $6,028.98.

Learn more about Compound Interest here:

https://brainly.com/question/14295570

#SPJ2

Select the reason that supports the following statement.

Answers

Answer:

B. Subtraction property of equality

Step-by-step explanation:

If you subtract angle 3 from both sides, then angle two and angle four must be equal. The property you would be using is the subtraction property of equality.

Answer:

Subtraction property of inequality

Step-by-step explanation:

See

<2+<3=<3+<4

As <3 is same on both sides We can cancel it out

The expression yields

<2=<4

The depth of the snow in my yard is normally distributed, with a mean of 2.5 inches and a standard deviation of 0.25 inches what value is one standard deviation above the mean

Answers

2.75 I think
because 2.50 plus 0.25 is 2.75

Don't you times by a thousand then times the answer
I got 60,000

Answers

Answer:

i think that's right

Step-by-step explanation:

Mr. Chow is buying rolls of paper for the school. This is a list of the suppliers, the size of their rolls, and their cost per roll. Supplier X sells 25 meter rolls for $18.98.
Supplier Y sells 30 meter rolls for $21.89.
Supplier Z sells 40 meter rolls for $29.59.
To the nearest cent, what is the cost per meter for a roll of paper from the LEAST expensive supplier?
A. $0.72 per meter
B. $0.73 per meter
O O O
C. $0.74 per meter
OD. $0.75 per meter
E. $0.76 per meter

Answers

Your answer is B.) $0.73 per meter

Answer:

B.

Step-by-step explanation:

I rounded to the nearest cent and got the cost of 73 cents per meter.

Use substitution to find Limit of StartStartFraction cosine (StartFraction pi x Over 4 EndFraction) OverOver x + 1 EndEndFraction as x approaches 3.

ANSWER: C

Answers

Answer:

C

Step-by-step explanation:

- square root 2 / 8

[tex]\bold{ \lim_{x \to 3} \frac{cos(\frac{\pi x}{4} )}{x+1} } = \bold{-\frac{\sqrt{2} }{8} }[/tex] , the correct answer is option (C)

What is limit?

"A limit is the value that a function approaches as the input approaches some value."

What is substitution in mathematics?

"Substitution is the process of replacing an algebraic letter with its value. "

Given: [tex]\lim_{x \to 3} \frac{cos(\frac{\pi x}{4} )}{x+1}[/tex]

By substitution the value of limit,

[tex]\lim_{x \to 3} \frac{cos(\frac{\pi x}{4} )}{x+1}[/tex]

= [tex]\frac{cos(\frac{\pi (3)}{4} )}{(3)+1}[/tex]

= [tex]\frac{cos(\frac{3\pi}{4} )}{4}[/tex]

= [tex]\frac{-\frac{1}{\sqrt{2} } }{4}[/tex]

= [tex]-\frac{1}{4\sqrt{2} }[/tex]

= [tex]\bold{-\frac{\sqrt{2}}{8} }[/tex]

Hence, option (C) is the correct answer.

Learn more about limit here:

brainly.com/question/1619243

#SPJ2

Name a pair of lines that are parallel

Answers

you didn't include the picture, so I cant see what the question is, sorry.

or if you mean any lines they just have to not intersect or combined at any point like the image I attached, that is parallel lines when they never touch.

What is the value of the expression 2[3(4^2+1)]-2^3?

Answers

Answer: 94

Step-by-step explanation: use photo math :)

Brainliest for correct answer

Answers

1.not a function 2. Not a function 3.function

Let f open parentheses x close parentheses equals ln open parentheses x squared close parentheses space space space a n d space space space g open parentheses x close parentheses equals square root of e to the power of 3 x end exponent end root. Find f o g open parentheses x close parentheses and its domain?

Answers

Answer:

[tex](f\ o\ g)(x) = 3x[/tex]

[tex]-\infty < x < \infty[/tex]

Step-by-step explanation:

See comment for right presentation of question

Given

[tex]f(x) = ln(x^2)[/tex]

[tex]g(x)=\sqrt{e^{3x}}[/tex]

Solving (a): (f o g)(x)

This is calculated as:

[tex](f\ o\ g)(x) = f(g(x))[/tex]

We have:

[tex]f(x) = ln(x^2)[/tex]

[tex]f(g(x)) = \ln((g(x))^2)[/tex]

Substitute: [tex]g(x)=\sqrt{e^{3x}}[/tex]

[tex]f(g(x)) = \ln(\sqrt{e^{3x}})^2[/tex]

Evaluate the square

[tex]f(g(x)) = \ln(e^{3x})[/tex]

Using laws of natural logarithm:

[tex]\ln(e^{ax}) = ax[/tex]

So:

[tex]f(g(x)) = \ln(e^{3x})[/tex]

[tex]f(g(x)) = 3x[/tex]

Hence:

[tex](f\ o\ g)(x) = 3x[/tex]

Solving (b): The domain

We have:

[tex]f(g(x)) = 3x[/tex]

The above function has does not have any undefined points and domain constraints.

Hence, the domain is: [tex]-\infty < x < \infty[/tex]

pls helppppp it's hard :(​

Answers

Answer:

The answer is A .

Step-by-step explanation:

Good luck ;)

Quincy is having a sale at his store. He is selling a table for $450. If the sales tax is 8.5%, then how much will a customer need to pay in taxes if they buy the table?

Answers

Answer:

488.25

Step-by-step explanation:

h = -16t ^2 + 38t + 5.

Answers

Answer:

for rewrite in vertex form is -16(t-19/16)^2 + 441/16

for rewrite in intercept form is h= -16(t-5/2)(t+1/8)

Step-by-step explanation:

Find the difference 0.2−0.05

Answers

Answer:

0.15

Step-by-step explanation:

:))))))))))))))))))))

27
The diagram shows a right-angled triangle and a quarter circle.
D
9 m
А
6 m
B.
The right-angled triangle ABC has angle ABC = 90°
The quarter circle has centre C and radius CB.
Work out the area of the quarter circle.
Give your answer correct to 3 significant figures.
You must show all your working.

Answers

Answer:

A  ≈ 35.3 units²

Step-by-step explanation:

Calculate the radius CB using Pythagoras' identity in the right triangle.

CB² + AB² = AC²

CB² + 6² = 9²

CB² + 36 = 81 ( subtract 36 from both sides )

CB²  = 45 = r²

Then area of quarter circle is

A = [tex]\frac{1}{4}[/tex] × πr² = [tex]\frac{1}{4}[/tex] × π × 45 ≈ 35.3

graph
[tex]y = - 2 - {e}^{ - \frac{ - x}{2} } [/tex]
what are the transformations??​

Answers

Graph attached

Y intercept is at -3

Miss Patterson is getting married soon. She would like to hire a professional who will
come to her house and do hair and makeup for the bridal party on her wedding day.
She has narrowed it down to two possible choices. A woman from Burlington Salon
charges an initial fee of $30 and then an additional $20 per hour. Alternatively, a team
from Beauty by Dirk offers a free consultation and then charges a higher hourly rate of
$30. If it takes a certain amount of time to finish everyone's hair and makeup, Miss
Patterson will end up paying the same amount either way. How much would Miss
Patterson end up paying?

a) Write a system of linear equations to represent this situation.

b) Graph that system (assume the graph is increasing by 10s rather than 1s).

C) Find the solution and explain what it means.

Answers

Answer:The first team

Step-by-step explanation:The first team charges least and 30 dollers a hour is alot if the time gets 5-10 hours.

Your welcome!

How would you describe a linear learning curve. Choose the best available answer. for each unit of time the performance improves a proportional amount for each unit of time the performance decreases a proportional amount the performer displays rapid initial improvement followed by a decline in the rate of improvement the performer displays little initial improvement followed a rapid increase in the rate of improvement none of the above

Answers

Answer:

My answer would be; the performer displays rapid initial improvement followed by a decline in the rate of improvement.

Step-by-step explanation:

Usually a learning curve means that struggles are occurring after time of understanding and if we were creating a linear graph of this learning curve, it would be decreasing.

Question Help
Fund A sells for $15 a share and has a 3-year average annual return of $0.25 a share. The beta value is 1.07.
Fund B sells for $17 a share and has a 3-year average annual return of $0.91 a share. The beta value is 0.77.
Jonquin wants to spend no more than $3600 investing in these two funds, but he wants to obtain at least $80
in annual revenue. Jonquin also wants to minimize the risk. Determine the number of shares of each fund that
Jonquin should buy.
Set up the linear programming problem. Let a represent the number of shares in Fund A, b represent the
number of shares in Fund B, and z represent the total beta value.
Minimize
z=0
subject to
15a + 17b
0.25a +0.91b
a 20, b20.
(Use integers or decimals for any numbers in the expressions. Do not include the $ symbol in your answers.)

Answers

Answer:

sana all may jowa jowain na kita

express 110111 base two to base ten​

Answers

Answer:

zh

Step-by-step explanation:

39
x
27
Solve for x round to nearest 10th

Answers

Answer:

526.50

Step-by-step explanation:

39 x 27 = 1,053

1,053 divided by 2 = 526.5

rounded to the nearest tenth would be 526.50

(im not sure if its correct)

Answer:

whats the question

Step-by-step explanation:

Round .67832947 to the nearest thousandths (three decimal places)

Answers

.678 is your answer

What is the sum of just the fraction parts of 1 5/8 and 1 6/8?

Answers

Answer:

Just the fraction parts: 11/8 or 1 3/8

Step-by-step explanation:

1 5/8 + 1 6/8 = 3 3/8

Hopefully this helps :3 sorry if wrong :( Plz mark brainiest if correct :D Your bootiful/handsome! Have a great day luv <3

-Bee~

Other Questions
A culture of bacteria has an initial population of 72000 bacteria and doubles every 10hours. Using the formula P = Po - 2 , where P is the population after t hours, Pois the initial population, t is the time in hours and d is the doubling time, what is thepopulation of bacteria in the culture after 17 hours, to the nearest whole number? What is the value of x in this equation?-4x + 8 = 42 James wants to purchase a car.The purchase price of the car is $27,000.00 The dealership will allow him to pay cash orfinance the car with a down payment of $5,000.00 and 60 monthly payments of$525.00. James chooses to finance the car.Answer the following questions:a) What is the amount financed?b) What is the total amount of monthly payments?c} What is the amount of the finance charge? im stuck on the first 2 answers the average of 6 members to 2 8 9 x 7 and 12 is 7 find the value of x The measure of ADB is 162. What is the measure of Darcy is a cognitive neuroscience graduate student. While thinking about the direction of her research, she tries to decide which theoretical perspective on personality makes the most sense to her. If she emphasizes the importance of gene-environment interactions, then she will likely focus on the _____ perspective, but if she emphasizes the importance of how mental processes affect personality, then she will likely focus on _____. 100 POINTS PLEASE DONT COPY FROM ONLINE write a paragraph about who is being affected from the enduring issue (impact of Industrialization) What is information cycle Two angles, m The m ABC is 54 The m JKL is (8x)".What is the value of x? 2 positive integers have hcf of 12 and their product as 6336 number of pairs possible for the number is In the South during the 19th century, poor white farmers: A. usually worked for plantation owners. B. had no legal rights, like enslaved people. O C. sometimes owned slaves. O D. often joined with other poor whites to run a plantation together. find the valu of x and y Can you pls explain why too Grayson ran 2 1/2 miles around the track .each lap is 5/6 of a mile .how many laps did Grayson run? Find the circumference of a circle that has a radius of 36 inches PLEASE HELP FAST!!!How does the dialogue in paragraph 24 contribute to the humor of the scene? In the story fourteen in commonlit PLEASE HELP IM TIMED ON THIS!!!!! After simplifying, what will x be equivalent to?(a^4)^4=a^x A chainsaw needs a 50:1 gas:oil ratio, how much oil do you need if you add 3 gallons of gas? no links!! links will be reported Two charged objects have a repulsive force of .080 N. If the charge of both of the objects is doubled, then what is the new force?